Gönderen Konu: Genç Balkan Matematik Olimpiyatı 2022 Soru 1  (Okunma sayısı 1725 defa)

Çevrimdışı matematikolimpiyati

  • Geo-Maniac
  • ********
  • İleti: 1418
  • Karma: +3/-0
Genç Balkan Matematik Olimpiyatı 2022 Soru 1
« : Temmuz 02, 2022, 02:14:07 öö »
                        $11ab \leq a^3-b^3 \leq 12ab$

koşullarını sağlayan tüm $(a,b)$ pozitif tam sayı ikililerini bulunuz.
« Son Düzenleme: Ekim 26, 2023, 03:21:31 ös Gönderen: Lokman Gökçe »

Çevrimdışı Metin Can Aydemir

  • G.O Genel Moderator
  • Geo-Maniac
  • ********
  • İleti: 1139
  • Karma: +9/-0
Ynt: Genç Balkan Matematik Olimpiyatı 2022 Soru 1
« Yanıtla #1 : Ekim 15, 2023, 12:28:36 ös »
$a>b$ olduğu açıktır. $k$ pozitif tamsayısı için $a=b+k$ yazalım. Bu durumda $$11(b^2+bk)\leq 3b^2k+3bk^2+k^3\leq 12(b^2+bk)\implies 0\leq (3k-11)b^2+(3k^2-11k)b+k^3\quad \text{ve}\quad (3k-12)b^2+(3k^2-12k)b+k^3\leq 0$$ elde edilir. Önce $k\geq 4$ durumunu inceleyelim. $$(3k-12)b^2+(3k^2-12k)b+k^3=(3k-12)\left(b^2+bk+\frac{k^2}{4}\right)+\frac{k^3}{4}+3k^2=3(k-4)\left(b+\frac{k}{2}\right)^2+\frac{k^3}{4}+3k^2\leq 0$$ çelişkisi elde edilir çünkü bariz bir şekilde ifade pozitiftir.

$k<4$ olmalıdır. $k=3$ ise $$0\leq -2b^2-6b+27\quad \text{ve}\quad -3b^2-9b+27\leq 0$$ elde edilir. İlk eşitsizliğin sağlanabilmesi için $b=1,2$ olmalıdır. $b=2$ için ikinci eşitsizlik de sağlanır. Buradan $\boxed{(a,b)=(5,2)}$ elde edilir.

$k=2$ ise $$0\leq -5b^2-10b+8\quad \text{ve}\quad -6b^2-12b+8\leq 0$$ elde edilir ancak ilk eşitsizlik hiçbir $b$ pozitif tamsayısı için sağlanmaz. Çözüm gelmez.

$k=1$ ise $$0\leq -8b^2-8b+1\quad \text{ve}\quad -9b^2-9b+1\leq 0$$ elde edilir benzer şekilde ilk eşitsizlik hiçbir $b$ pozitif tamsayısı için sağlanmaz. Çözüm gelmez.

Şartı sağlayan tek $(a,b)$ çifti $(5,2)$'dir.
« Son Düzenleme: Ekim 26, 2023, 03:21:39 ös Gönderen: Lokman Gökçe »
Gerçek hikayeler aslında söylenmeyenlerdir.

Çevrimdışı Metin Can Aydemir

  • G.O Genel Moderator
  • Geo-Maniac
  • ********
  • İleti: 1139
  • Karma: +9/-0
Ynt: Genç Balkan Matematik Olimpiyatı 2022 Soru 1
« Yanıtla #2 : Ekim 15, 2023, 01:43:33 ös »
$a>b$ olduğunu biliyoruz. Dolayısıyla $a^3-b^3=(a-b)(a^2+ab+b^2)>3ab(a-b)$ olacaktır. $12ab\geq a^3-b^3>3ab(a-b)$ olduğundan $4>a-b>0$, yani $a-b=1,2,3$ bulunur. Yerine koyularak, üstteki çözüme benzer şekilde eşitsizlikler çözülebilir.
Gerçek hikayeler aslında söylenmeyenlerdir.

Çevrimdışı Hüseyin Yiğit EMEKÇİ

  • Geo-Maniac
  • ********
  • İleti: 602
  • Karma: +2/-0
Ynt: Genç Balkan Matematik Olimpiyatı 2022 Soru 1
« Yanıtla #3 : Kasım 14, 2023, 06:07:53 ös »
Birazdan atacağın versiyonda orijinal sorudan farklı olarak eşitlik de bulunmaktadır.
''Uzman, çok dar bir alanda yapılabilecek tüm hataları yapmış kişidir.''   ~Niels Bohr

Çevrimdışı Hüseyin Yiğit EMEKÇİ

  • Geo-Maniac
  • ********
  • İleti: 602
  • Karma: +2/-0
Ynt: Genç Balkan Matematik Olimpiyatı 2022 Soru 1
« Yanıtla #4 : Kasım 14, 2023, 06:09:40 ös »
Versiyon 1
Aşağıdaki eşitsizliği sağlayan tüm $(a,b)$ pozitif tam sayı ikililerini


$$14ab\leq a^3-b^3\leq 15ab$$


bulunuz.
''Uzman, çok dar bir alanda yapılabilecek tüm hataları yapmış kişidir.''   ~Niels Bohr

Çevrimdışı Hüseyin Yiğit EMEKÇİ

  • Geo-Maniac
  • ********
  • İleti: 602
  • Karma: +2/-0
''Uzman, çok dar bir alanda yapılabilecek tüm hataları yapmış kişidir.''   ~Niels Bohr

Çevrimdışı Hüseyin Yiğit EMEKÇİ

  • Geo-Maniac
  • ********
  • İleti: 602
  • Karma: +2/-0
Ynt: Genç Balkan Matematik Olimpiyatı 2022 Soru 1
« Yanıtla #6 : Ocak 15, 2024, 12:07:49 öö »
Versiyon 1'de eşitlik durumu $(a,b)=(8,4)$ için sağlanmaktadır.
''Uzman, çok dar bir alanda yapılabilecek tüm hataları yapmış kişidir.''   ~Niels Bohr

 


Sitemap 1 2 3 4 5 6 7 8 9 10 11 12 13 14 15 16 17 18 19 20 21 22 23 24 25 26 27 28 29 30 31 32 33 34 35 36 37 
SimplePortal 2.3.3 © 2008-2010, SimplePortal